You are on page 1of 5

Answers and Explanations

1 a 2 b 3 a 4 b 5 c 6 d 7 a 8 d 9 b 10 b
11 d 12 d 13 a 14 c 15 c 16 d 17 a 18 c 19 d 20 c
21 a 22 a 23 d 24 c 25 a 26 c 27 c 28 a 29 d 30 b
31 c 32 c 33 b 34 d 35 d 36 a 37 b 38 d 39 d 40 d
41 d 42 c 43 c 44 d 45 a 46 c 47 c 48 b 49 b 50 a
51 c 52 b 53 c 54 d 55 d 56 a 57 d 58 a 59 a 60 d
61 c 62 d 63 a 64 d 65 b 66 a 67 d 68 a

1. a (ab)2 = ccb, the greatest possible value of ‘ab’ to be


first 7 terms is the middle term which is the fourth
31. Since 312 = 961 and since ccb > 300, 300 < ccb <
term. Hence, it is one more than the previous average.
961, so 18 < ab < 31. So the possible value of ab
which statisfies (ab)2 = ccb is 21. So 212 = 441,
11. d Numbers which are divisible by 3 (between 100 and
∴ a = 2, b = 1, c = 4. 200) are 33. Numbers which are divisible by 21, i.e.
LCM of 7 and 3 (between 100 and 200) are 5. Out of
2. b Note: 342 = 73 – 1. On further simplification we get, the 33 numbers divisible by 3, 17 are even and 16 are
odd. Out of the 5 numbers divisible by 7, three are
(73 )28 34328 (342 + 1)
28
= = = odd. Hence, the number of odd numbers divisible by 3
342 342 342 but not by 7 is (16 – 3) = 13.
342 N + 1 1
= = 12. d Take any three odd and positive numbers and check
342 342 this out.
Hence, remainder = 1, optiion (b).
13. a There is only one 5 and one 2 in the set of prime
3. a Use the method of simulation, viz. take any sample numbers. Hence, there would be only one zero at the
values of x and verify that n is both odd as well as a end of the resultant product.
perfect square.
14. c If the sides of the triangle are a, b and c, then
4. b If there are n numbers, the function h has to be a + b > c. Given a + b + c = 14.
performed one time less. Then the sides can be (4, 4, 6), (5, 5, 4), (6, 5, 3)
and (6, 6, 2). Hence, four triangles are possible.
5. c The square root is 11111111.
15. c N = 1421 × 1423 × 1425. When divided by 12, it shall
6. d None of the statements are true.
(1416 + 5 ) × (1416 + 7 ) × (1416 + 9 )
look like  .
7. a S1 remains the same, but S2 changes. 12
Now the remainder will be governed by the term
8. d x must be equal to the greatest difference in the value 5 × 7 × 9, which when divided by 12 leaves the
of numbers of S1 and S2. remainder 3.

a1 a2 16. d Let r be the remainder. Then 34041 – r and 32506 – r


9. b 99 × D = a1a2. Hence, D= . So D must be multiplied are perfectly divisible by n. Hence, their difference
99
should also be divisible by the same.
by 198 as 198 is a multiple of 99. (34041 – r) – (32506 – r) = 1535, which is divisible by
only 307.
10. b Use any 7 consecutive numbers to check the
answers. 17. a Each term has to be either 1 or –1.

n=
(1 + 2 + 3 + 4 + 5 ) = 3 , average of 7 integers is
Hence, if the sum of n such terms is 0, then n is even.
5
18. c There are two possible cases. The number 9 comes

k=
(1 + 2 + 3 + 4 + 5 + 6 + 7 ) = 4 . at the end, or it comes at position 4, 5, or 6.
For the first case, the number would look like:
7
So k = n + 1. 9
635 ... ... 9.
Alternately, the average of the first 5 terms is the 674
middle term which is third term, and the average of the In both these cases, the blanks can be occupied by

QA / CAT / Number System Page 9


any of the available 9 digits (0, 1, 2, ..., 8). 25. a The product of 44 and 11 is 484.
Thus, total possible numbers would be If base is x, then 3411
2 × (9 × 9 × 9) = 1458. For the second case, the
number 9 can occupy any of the given position 4, 5, or = 3x 3 + 4 x 2 + 1x1 + 4 × x 0 = 484
6, and there shall be an odd number at position 7.
Thus, the total number of ways shall be ⇒ 3 x3 + 4 x2 + x = 480
2[3(9 × 9 × 4)] = 1944. Hence, answer is 3402. This equation is satisfied only when x = 5.
So base is 5.
19. d N can be written either (54 + 1)3 + (18 – 1)3 – 723 or In decimal system, the number 3111 can be written
(51+ 4 )3 + 173 – (68 + 4)3 . 3 × 53 + 1 × 52 + 1 × 51 + 1 × 50 = 406
The first form is divisible by 3, and the second by 17.

20. c 26. c Let the total number of pages in the book be n.


n
1 2 19 82
1 2 16 5 – 2
Let page number x be repeated. Then ∑ i + x = 1000
i =1
1 2 13 – 9
1 – 1 n(n + 1)
+ x = 1000
2
The answer is 1192.
n(n + 1)
Thus, ≤ 1000 gives n = 44
21. a Use the answer choices and the fact that: 2
Odd × Odd = Odd
n(n + 1)
Odd × Even = Even Since = 990 (for n = 44).
Even × Even = Even 2
Hence x = 10.
22. a Let the four-digit number be abcd.
a+b=c+d ... (i) 27. c Take a = b = c = d = 1.
b + d = 2(a + c) ... (ii)
a+d=c ... (iii) 28. a Let the highest number be n and x be the number
From (i) and (iii), b = 2d erased.
From (i) and (ii), 3b = 4c + d
n(n + 1)
⇒ 3(2d) = 4c + d –x
7 602 .
Then 2 = 35 =
⇒ 5d = 4c (n – 1) 17 17
5 Hence, n = 69 and x = 7 satisfy the above conditions.
⇒ c= d
4
Now d can be 4 or 8. 29. d a = b2 – b, b ≥ 4
But if d = 8, then c = 10 not possible. a2 – 2a = (b2 – b)2 – 2(b2 – b)
So d = 4 which gives c = 5. = (b2 – b)(b2 – b – 2)
Using different values to b ≥ 4 and we find that it is
23. d Let the number be x. divisible by 15, 20, 24.
Increase in product = 53x – 35x = 18x Hence all of these is the right answer.
⇒ 18x = 540 ⇒ x = 30
Hence new product = 53 × 30 = 1590. 30. b From II, b = 2d
Hence, b = 10, d = 5 or b = 4, d = 2
24. c The value of y would be negative and the value of x From III, e + a = 10 or e + a = 4
would be positive from the inequalities given in the From I, a + c = e or e – a = c
question. From III and I, we get 2e = 10 + c or 2e = 4 + c
Therefore, from (a), y becomes positive. The value of c
xy2 would be positive and will not be the minimum. ⇒ e=5+ ... (i)
2
From (b) and (c), x2y and 5xy would give negative
values but we do not know which would be the c
minimum. or e = 2 + ... (ii)
2
On comparing (a) and (c), we find that
From (i), we can take c = 2, 4, 6, 10.
x2 < 5x in 2 < x < 3.
For c = 2, e = 6
∴ x2 y > 5xy [Since y is negative.] c = 4, e = 7 (Not possible)
∴ 5xy would give the minimum value. c = 6, c = 8 (Not possible)
c = 10, e = 10 (Not possible since both c and e cannot
be 10)

Page 10 QA / CAT / Number System


From (ii), we have c = 2, 4, 6, 10.
For c = 2, e = 3 (Not possible)  9 27 36  HCF(9, 27, 36) = 9
38. d HCF  , ,  = LCM (2, 4, 5) 20
lb
c = 4, e = 4 (Not possible) 2 4 5 
c = 6, e = 5 (Possible) = Weight of each piece
c = 10, e = 7 (Not possible) Total weight = 18.45 lb
Considering the possibility from B that c = 6 and
e = 5 means e + a = 4 18.45 ×20
Maximum number of guests = = 41
⇒ a = –1 (Not possible) 9
Hence, only possibility is b = 10, d = 5, c = 2, e = 6.
e + a = 10 ⇒ a = 4 39. d 3(4(7x + 4) + 1) + 2 = 84x + 53
Therefore, remainder is 53.
31. c The last two digits can be 12, 16, 24, 32, 36, 52, 56,
and 64, i.e. 8 possibilites 40. d um + vm = w m
4
Remaining digits can be chosen in P3 = 24 ways.
u2 + v 2 = w 2
Hence, total number of such five-digit numbers Taking Pythagorean triplet 3, 4 and 5, we see
= 24 × 8 = 192. m < min (u, v, w)
Also 1' + 2' = 3' and hence m ≤ min (u, v, w)
32. c Total possible arrangements = 10 × 9 × 8
Now 3 numbers can be arranged among themselves
in 3! ways = 6 ways 41. d 76n – 66n
Given condition is satisfied by only 1 out of 6 ways. Put n = 1
Hence, the required number of arrangements
76 – 66 = (73 – 63 )(73 + 63 )
10 × 9 × 8
= = 120
6 This is a multiple of 73 – 63 = 127 and 73 + 63 = 559
and 7 + 6 = 13
33. b Check choices
Choice (b) 54 ⇒ S = (5 + 4)2 = 81 42. c Number of ways for single digit = 2
⇒ D – S = 81 – 54 = 27. Hence, the number = 54 2 digits = 2 × 3 = 6
3 digits = 2 × 3 × 3 = 18
34. d Let the number of gold coins = x + y 4 digits = 2 × 3 × 3 × 3 = 54
48(x – y) = X2 – Y2 5 digits = 2 × 3 × 3 × 3 × 3 = 162
48(x – y) = (x – y)(x + y) ⇒ x + y = 48 6 digits = 2 × 3 × 3 × 3 × 3 × 3 = 486
Hence the correct choice would be none of these. Total = 728

43. c There are 101 integers in all, of which 51 are even.


n2 + n From 100 to 200, there are 14 multiples of 7, of which
35. d 575 = –x
2 7 are even.
There are 11 multiples of 9, of which 6 are even.
1150 = n2 + n – 2x But there is one integer (i.e. 126) that is a multiple of
both 7 and 9 and also even.
n(n + 1) ≥ 1150
Hence the answer is (51 – 7 – 6 + 1) = 39
n2 + n ≥ 1150
The smallest value for it is n = 34. 44. d Since the last digit in base 2, 3 and 5 is 1, the number
For n = 34 should be such that on dividing by either 2, 3 or 5 we
should get a remainder 1. The smallest such number is
40 = 2x ⇒ x = 20
31. The next set of numbers are 61, 91.
Among these only 31 and 91 are a part of the answer

(2 ) choices.
64
4
36. a = (17 – 1)64 = 17n + ( −1)64 = 17n + 1
Among these, (31)10 = (11111)2 = (1011)3 = (111)5
Hence, remainder = 1
Thus, all three forms have leading digit 1.
37. b Because each word is lit for a second, Hence the answer is 91.

5 17 41   7 21 49  45. a Solution cannot be found by using only Statement A


LCM  + 1, + 1, + 1 = LCM  , , 
2 4 8  2 4 8  since b can take any even number 2, 4, 6….
But we can arrive at solution by using statement B
LCM(7, 21, 49) 49 × 3 alone.
= = 73.5 s
HCF ( 2, 4, 8) 2 If b > 16, say b = 17
Hence 2 44 < (16 + 1)11
244 < (24 + 1)11

QA / CAT / Number System Page 11


46. c If y = 2 (it cannot be 0 or 1), then x can take 1 value
and z can take 2 values. 41
=4
Thus with y = 2, a total of 1 × 2 = 2 numbers can be 6
formed. With y = 3, 2 × 3 = 6 numbers can be formed.
Similarly checking for all values of y from 2 to 9 and 42
adding up we get the answer as 240. =4
6
47. c The best way to do this is to take some value and 43
verify. =4
6
1
E.g. 2, and 1. Thus, n = 3 and the sum of the three
2 44
=4
numbers = 3.5. 6
Thus options 1, 2 and 4 get eliminated. Hence, any power of 4 when divided by 6 leaves a
remainder of 4.
Alternative method:
Let the n positive numbers be a1, a2, a3 … an
a1, a2, a 3 … an = 1 55. d Let n = 6
We know that AM ≥ GM Therefore, n = 6 ≈ 2.4
1 Therefore, divisors of 6 are 1, 2, 3.
Hence (a1 + a 2 + a3 + … + an ) ≥ (a1a 2 …an )1/ n
n If we take 2 as divisor, then n > 2 > 1.
or (a1 + a2 + a3 + … an) ≥ n Statement A is true.
If we take 3 as divisor, then 6 > 3 > 2.4, i.e. n > 3 > n.
48. b From 12 to 40, there are 7 prime number, i.e. 13, 17,
Therefore, statement B is true.
19, 23, 29, 31, 37, which is not divisible by (n–1)!
56. a As any prime number greater than 3 can be expressed
49. b MDCCLXXXVII = 1000 + 500 + 100 + 100 + 50 + 10 + 10
+ 10 + 5 + 1 + 1 = 1787 in the form 6n ± 1 , minimum difference between three
consecutive prime numbers will be 2 and 4. The values
50. a MCMXCIX = 1000 + (1000 – 100) + (100 – 10) + (10 – that satisfy the given conditions are only 3, 5 and 7,
1) = 1000 + 900 + 90 + 9 = 1999 i.e. only one set is possible.

51. c (I) MCMLXXV = 1000 + (1000 – 100) + 50 + 10 + 10 57. d a = 6b = 12c and 2b = 9d = 12e.
+ 5 = 1975 Dividing the first equations by 12 and second by 36,
(II) MCMXCV = 1000 + (1000 – 100) + (100 – 10) + 5
a b c b d e
= 1995 we get = = and = =
(III) MVD = 1000 + (500 – 5) = 1495 12 2 1 18 4 3
(IV) MVM = 1000 + (1000 – 5) = 1995 a b c b d e
Therefore, the answer is (II) and (IV), i.e. option (c). i.e. = = and = =
108 18 9 18 4 3
52. b Such numbers are 10, 17, …, 94. a b c d e
These numbers are in AP. There are 13 numbers. ∴ = = = =
108 18 9 4 3
10 + 94 ∴ a : b : c : d : e = 108 : 18 : 9 : 4 : 3.
∴ Sum = × 13
2
c 9
= 52 × 13 = 676 ∴ = is not an integer.
d 4
53. c Total codes which can be formed = 9 × 9 = 81.
(Distinct digit codes) 58. a There will be an increase of 6 times.
The digits which can confuse are 1, 6, 8, 9, from No. of members s 1 will be in A.P.
these digit we can form the codes = 4 × 3 = 12 On July 2nd , 2004, s1 will have n + 6 b members
Out of these 12 codes two numbers 69 and 96 will not = n + 6 × 10.5 n
create confusion. = 64n
Therefore, (12 – 2) = 10 codes will create a confusion. No. of members in s 2 will be in G.P
Therefore, total codes without confusion On July 2nd, 2004 Number of members in s2
= 81 – 10 = 71. = nr6
They are equal, Hence 64 n = nr6
496
54. d Remainder when ⇒ 64 = r 6 ⇒ r = 2
6
Let’s come down to basic property of dividing the 59. a We have
power of 4 by 6, i.e. (1) 1010 < n < 1011
(2) Sum of the digits for 'n' = 2

Page 12 QA / CAT / Number System


Clearly- 64. d If p = 1! = 1
(n)min = 10000000001 (1 followed by 9 zeros and Then p + 2 = 3 when divided by 2! remainder will be
finally 1) 1.
Obviously, we can form 10 such numbers by shifting If p = 1! + 2 × 2! = 5
'1' by one place from right to left again and again. Then p + 2 = 7 when divided by 3! remainder is still
Again, there is another possibility for 'n' 1.
n = 20000000000 Hence p = 1! + (2 × 2!) + (3 × 3!) + … + (10 × 10!)
So finally : No. of different values for n = 10 + 1 = 11 when divided by 11! leaves remainder 1
ans.
Alternative method:
P = 1 + 2.2! + 3.3!+ ….10.10!
1
60. d y= = (2 –1)1! + ( 3 – 1)2! + (4 – 1)3! + ….(11 – 1)10!
1 =2! – 1! + 3! – 2! + ….. 11! –10!
2+
3+ y = 1 + 11!
Hence the remainder is 1.
3+y
⇒y=
7 + 2y 65. b Let A = 100 x + 10y + z
⇒ B = 100z + 10y + x
⇒ 2y 2 + 6y – 3 = 0 B - A = 99(z - x)
For B - A to be divided by 7, z - x has to be divisible
–6 ± 36 + 24 by 7. Only possibility is z = 9, x = 2.
⇒y=
4 Biggest number A can be 299
Option b.
–6 ± 60 –3 ± 15
= =
4 2

Since 'y' is a +ve number, therefore:


66. a ((30) )
4 680

Hence the right most non-zero digit is 1.


15 –3
y= ans.
2 67. d 10 < n < 1000
Let n is two digit number.
n = 10a + b ⇒ pn = ab, sn = a + b
61. c 1523 = (19 – 4)23 = 19x + (–4)23 where x is a natural Then ab + a + b = 10a + b
number. ⇒ ab = 9a ⇒ b = 9
There are 9 such numbers 19, 29, 33, … 99
2323 = (19 + 4)23 = 19y + (4)23 where y is a natural Then Let n is three digit number
number. ⇒ n = 100a + 10b + c ⇒ pn = abc, sn= a + b + c
then abc + a + b + c = 100a + 10b + c
1523 + 2323 = 19 ( x + y ) + 423 + (–4)23 ⇒ abc = 99a + 9b
= 19 ( x +y) b
⇒ bc = 99 + 9
a
3065 – 29 65 But the maximum value for bc = 81
62. d >1 And RHS is more than 99. Hence no such number is
3064 + 2964 possible.
as 3065 – 2965 > 3064 + 2964 Hence option d.
3064 (30 – 1) > 2964 (29 + 1)
3064 × 29 > 2964 × 30 68. a The 100th and 1000th position value will be only 1 .
3063 > 2963 Now the possibility of unit and tens digits are
Hence option d. (1, 3), (1, 9), (3, 1), (3, 7), (5, 5), (7, 3), (7, 9), (9, 1),
(9, 7).
63. a x = 163 + 173 + 183 + 193 is even number
Therefore 2 divides x.
a3 + b3 = (a + b) (a2 – ab – b2)
⇒ a + b always divides
Therefore 163 + 193 is divisible by 35
183 + 173 is divisible by 35
Hence x is divisible by 70.
Hence option a.

QA / CAT / Number System Page 13

You might also like